There is only s̶i̶x̶t̶y one place he can be












26












$begingroup$


This puzzle is not mine, although I don't have any reference as to where it originated from. I discovered it when I was a student, some fifteen years ago. It has been among my favourite chess puzzles since then. Could be very well known or a duplicate, but I couldn't find it in the database.



The following position is legal, except that the White King (and the White King only) is not displayed. Find it.



                  










share|improve this question









$endgroup$








  • 3




    $begingroup$
    It appears this problem may have originated from The Chess Mysteries of the Arabian Knights by Raymond Smullyan.
    $endgroup$
    – noedne
    Mar 26 at 16:05






  • 1




    $begingroup$
    Related on Chess.SE: How does one know if a position is not reachable from a series of legal moves from the starting position?, showing some examples of this kind of puzzle (retrograde analysis), including this particular one.
    $endgroup$
    – Andrew T.
    Mar 27 at 10:20
















26












$begingroup$


This puzzle is not mine, although I don't have any reference as to where it originated from. I discovered it when I was a student, some fifteen years ago. It has been among my favourite chess puzzles since then. Could be very well known or a duplicate, but I couldn't find it in the database.



The following position is legal, except that the White King (and the White King only) is not displayed. Find it.



                  










share|improve this question









$endgroup$








  • 3




    $begingroup$
    It appears this problem may have originated from The Chess Mysteries of the Arabian Knights by Raymond Smullyan.
    $endgroup$
    – noedne
    Mar 26 at 16:05






  • 1




    $begingroup$
    Related on Chess.SE: How does one know if a position is not reachable from a series of legal moves from the starting position?, showing some examples of this kind of puzzle (retrograde analysis), including this particular one.
    $endgroup$
    – Andrew T.
    Mar 27 at 10:20














26












26








26


1



$begingroup$


This puzzle is not mine, although I don't have any reference as to where it originated from. I discovered it when I was a student, some fifteen years ago. It has been among my favourite chess puzzles since then. Could be very well known or a duplicate, but I couldn't find it in the database.



The following position is legal, except that the White King (and the White King only) is not displayed. Find it.



                  










share|improve this question









$endgroup$




This puzzle is not mine, although I don't have any reference as to where it originated from. I discovered it when I was a student, some fifteen years ago. It has been among my favourite chess puzzles since then. Could be very well known or a duplicate, but I couldn't find it in the database.



The following position is legal, except that the White King (and the White King only) is not displayed. Find it.



                  







chess






share|improve this question













share|improve this question











share|improve this question




share|improve this question










asked Mar 26 at 15:35









Arnaud MortierArnaud Mortier

1,970724




1,970724








  • 3




    $begingroup$
    It appears this problem may have originated from The Chess Mysteries of the Arabian Knights by Raymond Smullyan.
    $endgroup$
    – noedne
    Mar 26 at 16:05






  • 1




    $begingroup$
    Related on Chess.SE: How does one know if a position is not reachable from a series of legal moves from the starting position?, showing some examples of this kind of puzzle (retrograde analysis), including this particular one.
    $endgroup$
    – Andrew T.
    Mar 27 at 10:20














  • 3




    $begingroup$
    It appears this problem may have originated from The Chess Mysteries of the Arabian Knights by Raymond Smullyan.
    $endgroup$
    – noedne
    Mar 26 at 16:05






  • 1




    $begingroup$
    Related on Chess.SE: How does one know if a position is not reachable from a series of legal moves from the starting position?, showing some examples of this kind of puzzle (retrograde analysis), including this particular one.
    $endgroup$
    – Andrew T.
    Mar 27 at 10:20








3




3




$begingroup$
It appears this problem may have originated from The Chess Mysteries of the Arabian Knights by Raymond Smullyan.
$endgroup$
– noedne
Mar 26 at 16:05




$begingroup$
It appears this problem may have originated from The Chess Mysteries of the Arabian Knights by Raymond Smullyan.
$endgroup$
– noedne
Mar 26 at 16:05




1




1




$begingroup$
Related on Chess.SE: How does one know if a position is not reachable from a series of legal moves from the starting position?, showing some examples of this kind of puzzle (retrograde analysis), including this particular one.
$endgroup$
– Andrew T.
Mar 27 at 10:20




$begingroup$
Related on Chess.SE: How does one know if a position is not reachable from a series of legal moves from the starting position?, showing some examples of this kind of puzzle (retrograde analysis), including this particular one.
$endgroup$
– Andrew T.
Mar 27 at 10:20










2 Answers
2






active

oldest

votes


















22












$begingroup$

The white king must be on square




c3




Explanation:




First, we determine which side is to move. Unless the white king blocks the a4-d1 diagonal, the black king is in check. The king can't be on c2 since it would attack the black king; on b3, White would be to move but there's no way this double-check could have happened in a real game; there's no way to clear both the b3-d5 diagonal and the b3-b5 file with one move from the black rook or the black bishop. Black simply couldn't have played Bb4-d5 or Rc4-b5. So the black king is in check and White made the last move.




So, what was the last move?




The bishop can't have moved along the d1-a4 diagonal, because it would give check on c2 and d3 as well. So it must be a discovered check, caused by a move from the white king, from b3 to another square.




But ...




doesn't this lead to the same problem as before? On b3, the White king is still in double check. No, because there is a move which simultaneously clears the b3-d5 diagonal and the b3-b5 file, but it requires extra pieces: an en-passant capture from b4 to c3, capturing a pawn on c4. Since we have an extra (half-)move compared to the situation above, we can use that move to get rid of the black pawn.




So,




if White played 1. c2-c4, Black can play 1... b4xc3 e.p., White responds with 2. Kxc3 and we're in the diagram in the question with the White king on c3.







share|improve this answer











$endgroup$













  • $begingroup$
    ... but it doesn't include an explanation why this is the only possible solution (which took me a while to write down...)
    $endgroup$
    – Glorfindel
    Mar 26 at 15:59






  • 3




    $begingroup$
    @ArnaudMortier Glorfindel's answer eliminates other possibilities.
    $endgroup$
    – noedne
    Mar 26 at 15:59



















13












$begingroup$


The last move in this position was Bd5+. White blocked the check with c4, black gave double check by capturing en passant bxc, and the king took the pawn on c3.
chessboard







share|improve this answer









$endgroup$













  • $begingroup$
    Um... to me, it looks like rot13(gur cnja vf ba p2)... :P
    $endgroup$
    – EKons
    Mar 26 at 15:49










  • $begingroup$
    I've ROT13-ed part of the comment so that it doesn't appear as an immediate spoiler. Check the image you've posted.
    $endgroup$
    – EKons
    Mar 26 at 15:51












  • $begingroup$
    That was super quick. Did you know the problem before? Do you know where it is from?
    $endgroup$
    – Arnaud Mortier
    Mar 26 at 15:54










  • $begingroup$
    Your answer says that "rot13(gur xvat gbbx gur cnja ba p3)", however that square looks empty, unless there's actually more behind the image.
    $endgroup$
    – EKons
    Mar 26 at 15:54










  • $begingroup$
    @EKons After the black pawn captures en passant, it is on c3 for the white king to capture.
    $endgroup$
    – noedne
    Mar 26 at 15:56












Your Answer





StackExchange.ifUsing("editor", function () {
return StackExchange.using("mathjaxEditing", function () {
StackExchange.MarkdownEditor.creationCallbacks.add(function (editor, postfix) {
StackExchange.mathjaxEditing.prepareWmdForMathJax(editor, postfix, [["$", "$"], ["\\(","\\)"]]);
});
});
}, "mathjax-editing");

StackExchange.ready(function() {
var channelOptions = {
tags: "".split(" "),
id: "559"
};
initTagRenderer("".split(" "), "".split(" "), channelOptions);

StackExchange.using("externalEditor", function() {
// Have to fire editor after snippets, if snippets enabled
if (StackExchange.settings.snippets.snippetsEnabled) {
StackExchange.using("snippets", function() {
createEditor();
});
}
else {
createEditor();
}
});

function createEditor() {
StackExchange.prepareEditor({
heartbeatType: 'answer',
autoActivateHeartbeat: false,
convertImagesToLinks: false,
noModals: true,
showLowRepImageUploadWarning: true,
reputationToPostImages: null,
bindNavPrevention: true,
postfix: "",
imageUploader: {
brandingHtml: "Powered by u003ca class="icon-imgur-white" href="https://imgur.com/"u003eu003c/au003e",
contentPolicyHtml: "User contributions licensed under u003ca href="https://creativecommons.org/licenses/by-sa/3.0/"u003ecc by-sa 3.0 with attribution requiredu003c/au003e u003ca href="https://stackoverflow.com/legal/content-policy"u003e(content policy)u003c/au003e",
allowUrls: true
},
noCode: true, onDemand: true,
discardSelector: ".discard-answer"
,immediatelyShowMarkdownHelp:true
});


}
});














draft saved

draft discarded


















StackExchange.ready(
function () {
StackExchange.openid.initPostLogin('.new-post-login', 'https%3a%2f%2fpuzzling.stackexchange.com%2fquestions%2f81057%2fthere-is-only-s%25cc%25b6i%25cc%25b6x%25cc%25b6t%25cc%25b6y-one-place-he-can-be%23new-answer', 'question_page');
}
);

Post as a guest















Required, but never shown

























2 Answers
2






active

oldest

votes








2 Answers
2






active

oldest

votes









active

oldest

votes






active

oldest

votes









22












$begingroup$

The white king must be on square




c3




Explanation:




First, we determine which side is to move. Unless the white king blocks the a4-d1 diagonal, the black king is in check. The king can't be on c2 since it would attack the black king; on b3, White would be to move but there's no way this double-check could have happened in a real game; there's no way to clear both the b3-d5 diagonal and the b3-b5 file with one move from the black rook or the black bishop. Black simply couldn't have played Bb4-d5 or Rc4-b5. So the black king is in check and White made the last move.




So, what was the last move?




The bishop can't have moved along the d1-a4 diagonal, because it would give check on c2 and d3 as well. So it must be a discovered check, caused by a move from the white king, from b3 to another square.




But ...




doesn't this lead to the same problem as before? On b3, the White king is still in double check. No, because there is a move which simultaneously clears the b3-d5 diagonal and the b3-b5 file, but it requires extra pieces: an en-passant capture from b4 to c3, capturing a pawn on c4. Since we have an extra (half-)move compared to the situation above, we can use that move to get rid of the black pawn.




So,




if White played 1. c2-c4, Black can play 1... b4xc3 e.p., White responds with 2. Kxc3 and we're in the diagram in the question with the White king on c3.







share|improve this answer











$endgroup$













  • $begingroup$
    ... but it doesn't include an explanation why this is the only possible solution (which took me a while to write down...)
    $endgroup$
    – Glorfindel
    Mar 26 at 15:59






  • 3




    $begingroup$
    @ArnaudMortier Glorfindel's answer eliminates other possibilities.
    $endgroup$
    – noedne
    Mar 26 at 15:59
















22












$begingroup$

The white king must be on square




c3




Explanation:




First, we determine which side is to move. Unless the white king blocks the a4-d1 diagonal, the black king is in check. The king can't be on c2 since it would attack the black king; on b3, White would be to move but there's no way this double-check could have happened in a real game; there's no way to clear both the b3-d5 diagonal and the b3-b5 file with one move from the black rook or the black bishop. Black simply couldn't have played Bb4-d5 or Rc4-b5. So the black king is in check and White made the last move.




So, what was the last move?




The bishop can't have moved along the d1-a4 diagonal, because it would give check on c2 and d3 as well. So it must be a discovered check, caused by a move from the white king, from b3 to another square.




But ...




doesn't this lead to the same problem as before? On b3, the White king is still in double check. No, because there is a move which simultaneously clears the b3-d5 diagonal and the b3-b5 file, but it requires extra pieces: an en-passant capture from b4 to c3, capturing a pawn on c4. Since we have an extra (half-)move compared to the situation above, we can use that move to get rid of the black pawn.




So,




if White played 1. c2-c4, Black can play 1... b4xc3 e.p., White responds with 2. Kxc3 and we're in the diagram in the question with the White king on c3.







share|improve this answer











$endgroup$













  • $begingroup$
    ... but it doesn't include an explanation why this is the only possible solution (which took me a while to write down...)
    $endgroup$
    – Glorfindel
    Mar 26 at 15:59






  • 3




    $begingroup$
    @ArnaudMortier Glorfindel's answer eliminates other possibilities.
    $endgroup$
    – noedne
    Mar 26 at 15:59














22












22








22





$begingroup$

The white king must be on square




c3




Explanation:




First, we determine which side is to move. Unless the white king blocks the a4-d1 diagonal, the black king is in check. The king can't be on c2 since it would attack the black king; on b3, White would be to move but there's no way this double-check could have happened in a real game; there's no way to clear both the b3-d5 diagonal and the b3-b5 file with one move from the black rook or the black bishop. Black simply couldn't have played Bb4-d5 or Rc4-b5. So the black king is in check and White made the last move.




So, what was the last move?




The bishop can't have moved along the d1-a4 diagonal, because it would give check on c2 and d3 as well. So it must be a discovered check, caused by a move from the white king, from b3 to another square.




But ...




doesn't this lead to the same problem as before? On b3, the White king is still in double check. No, because there is a move which simultaneously clears the b3-d5 diagonal and the b3-b5 file, but it requires extra pieces: an en-passant capture from b4 to c3, capturing a pawn on c4. Since we have an extra (half-)move compared to the situation above, we can use that move to get rid of the black pawn.




So,




if White played 1. c2-c4, Black can play 1... b4xc3 e.p., White responds with 2. Kxc3 and we're in the diagram in the question with the White king on c3.







share|improve this answer











$endgroup$



The white king must be on square




c3




Explanation:




First, we determine which side is to move. Unless the white king blocks the a4-d1 diagonal, the black king is in check. The king can't be on c2 since it would attack the black king; on b3, White would be to move but there's no way this double-check could have happened in a real game; there's no way to clear both the b3-d5 diagonal and the b3-b5 file with one move from the black rook or the black bishop. Black simply couldn't have played Bb4-d5 or Rc4-b5. So the black king is in check and White made the last move.




So, what was the last move?




The bishop can't have moved along the d1-a4 diagonal, because it would give check on c2 and d3 as well. So it must be a discovered check, caused by a move from the white king, from b3 to another square.




But ...




doesn't this lead to the same problem as before? On b3, the White king is still in double check. No, because there is a move which simultaneously clears the b3-d5 diagonal and the b3-b5 file, but it requires extra pieces: an en-passant capture from b4 to c3, capturing a pawn on c4. Since we have an extra (half-)move compared to the situation above, we can use that move to get rid of the black pawn.




So,




if White played 1. c2-c4, Black can play 1... b4xc3 e.p., White responds with 2. Kxc3 and we're in the diagram in the question with the White king on c3.








share|improve this answer














share|improve this answer



share|improve this answer








edited Mar 26 at 16:07

























answered Mar 26 at 15:41









GlorfindelGlorfindel

14.3k45286




14.3k45286












  • $begingroup$
    ... but it doesn't include an explanation why this is the only possible solution (which took me a while to write down...)
    $endgroup$
    – Glorfindel
    Mar 26 at 15:59






  • 3




    $begingroup$
    @ArnaudMortier Glorfindel's answer eliminates other possibilities.
    $endgroup$
    – noedne
    Mar 26 at 15:59


















  • $begingroup$
    ... but it doesn't include an explanation why this is the only possible solution (which took me a while to write down...)
    $endgroup$
    – Glorfindel
    Mar 26 at 15:59






  • 3




    $begingroup$
    @ArnaudMortier Glorfindel's answer eliminates other possibilities.
    $endgroup$
    – noedne
    Mar 26 at 15:59
















$begingroup$
... but it doesn't include an explanation why this is the only possible solution (which took me a while to write down...)
$endgroup$
– Glorfindel
Mar 26 at 15:59




$begingroup$
... but it doesn't include an explanation why this is the only possible solution (which took me a while to write down...)
$endgroup$
– Glorfindel
Mar 26 at 15:59




3




3




$begingroup$
@ArnaudMortier Glorfindel's answer eliminates other possibilities.
$endgroup$
– noedne
Mar 26 at 15:59




$begingroup$
@ArnaudMortier Glorfindel's answer eliminates other possibilities.
$endgroup$
– noedne
Mar 26 at 15:59











13












$begingroup$


The last move in this position was Bd5+. White blocked the check with c4, black gave double check by capturing en passant bxc, and the king took the pawn on c3.
chessboard







share|improve this answer









$endgroup$













  • $begingroup$
    Um... to me, it looks like rot13(gur cnja vf ba p2)... :P
    $endgroup$
    – EKons
    Mar 26 at 15:49










  • $begingroup$
    I've ROT13-ed part of the comment so that it doesn't appear as an immediate spoiler. Check the image you've posted.
    $endgroup$
    – EKons
    Mar 26 at 15:51












  • $begingroup$
    That was super quick. Did you know the problem before? Do you know where it is from?
    $endgroup$
    – Arnaud Mortier
    Mar 26 at 15:54










  • $begingroup$
    Your answer says that "rot13(gur xvat gbbx gur cnja ba p3)", however that square looks empty, unless there's actually more behind the image.
    $endgroup$
    – EKons
    Mar 26 at 15:54










  • $begingroup$
    @EKons After the black pawn captures en passant, it is on c3 for the white king to capture.
    $endgroup$
    – noedne
    Mar 26 at 15:56
















13












$begingroup$


The last move in this position was Bd5+. White blocked the check with c4, black gave double check by capturing en passant bxc, and the king took the pawn on c3.
chessboard







share|improve this answer









$endgroup$













  • $begingroup$
    Um... to me, it looks like rot13(gur cnja vf ba p2)... :P
    $endgroup$
    – EKons
    Mar 26 at 15:49










  • $begingroup$
    I've ROT13-ed part of the comment so that it doesn't appear as an immediate spoiler. Check the image you've posted.
    $endgroup$
    – EKons
    Mar 26 at 15:51












  • $begingroup$
    That was super quick. Did you know the problem before? Do you know where it is from?
    $endgroup$
    – Arnaud Mortier
    Mar 26 at 15:54










  • $begingroup$
    Your answer says that "rot13(gur xvat gbbx gur cnja ba p3)", however that square looks empty, unless there's actually more behind the image.
    $endgroup$
    – EKons
    Mar 26 at 15:54










  • $begingroup$
    @EKons After the black pawn captures en passant, it is on c3 for the white king to capture.
    $endgroup$
    – noedne
    Mar 26 at 15:56














13












13








13





$begingroup$


The last move in this position was Bd5+. White blocked the check with c4, black gave double check by capturing en passant bxc, and the king took the pawn on c3.
chessboard







share|improve this answer









$endgroup$




The last move in this position was Bd5+. White blocked the check with c4, black gave double check by capturing en passant bxc, and the king took the pawn on c3.
chessboard








share|improve this answer












share|improve this answer



share|improve this answer










answered Mar 26 at 15:41









noednenoedne

8,23412364




8,23412364












  • $begingroup$
    Um... to me, it looks like rot13(gur cnja vf ba p2)... :P
    $endgroup$
    – EKons
    Mar 26 at 15:49










  • $begingroup$
    I've ROT13-ed part of the comment so that it doesn't appear as an immediate spoiler. Check the image you've posted.
    $endgroup$
    – EKons
    Mar 26 at 15:51












  • $begingroup$
    That was super quick. Did you know the problem before? Do you know where it is from?
    $endgroup$
    – Arnaud Mortier
    Mar 26 at 15:54










  • $begingroup$
    Your answer says that "rot13(gur xvat gbbx gur cnja ba p3)", however that square looks empty, unless there's actually more behind the image.
    $endgroup$
    – EKons
    Mar 26 at 15:54










  • $begingroup$
    @EKons After the black pawn captures en passant, it is on c3 for the white king to capture.
    $endgroup$
    – noedne
    Mar 26 at 15:56


















  • $begingroup$
    Um... to me, it looks like rot13(gur cnja vf ba p2)... :P
    $endgroup$
    – EKons
    Mar 26 at 15:49










  • $begingroup$
    I've ROT13-ed part of the comment so that it doesn't appear as an immediate spoiler. Check the image you've posted.
    $endgroup$
    – EKons
    Mar 26 at 15:51












  • $begingroup$
    That was super quick. Did you know the problem before? Do you know where it is from?
    $endgroup$
    – Arnaud Mortier
    Mar 26 at 15:54










  • $begingroup$
    Your answer says that "rot13(gur xvat gbbx gur cnja ba p3)", however that square looks empty, unless there's actually more behind the image.
    $endgroup$
    – EKons
    Mar 26 at 15:54










  • $begingroup$
    @EKons After the black pawn captures en passant, it is on c3 for the white king to capture.
    $endgroup$
    – noedne
    Mar 26 at 15:56
















$begingroup$
Um... to me, it looks like rot13(gur cnja vf ba p2)... :P
$endgroup$
– EKons
Mar 26 at 15:49




$begingroup$
Um... to me, it looks like rot13(gur cnja vf ba p2)... :P
$endgroup$
– EKons
Mar 26 at 15:49












$begingroup$
I've ROT13-ed part of the comment so that it doesn't appear as an immediate spoiler. Check the image you've posted.
$endgroup$
– EKons
Mar 26 at 15:51






$begingroup$
I've ROT13-ed part of the comment so that it doesn't appear as an immediate spoiler. Check the image you've posted.
$endgroup$
– EKons
Mar 26 at 15:51














$begingroup$
That was super quick. Did you know the problem before? Do you know where it is from?
$endgroup$
– Arnaud Mortier
Mar 26 at 15:54




$begingroup$
That was super quick. Did you know the problem before? Do you know where it is from?
$endgroup$
– Arnaud Mortier
Mar 26 at 15:54












$begingroup$
Your answer says that "rot13(gur xvat gbbx gur cnja ba p3)", however that square looks empty, unless there's actually more behind the image.
$endgroup$
– EKons
Mar 26 at 15:54




$begingroup$
Your answer says that "rot13(gur xvat gbbx gur cnja ba p3)", however that square looks empty, unless there's actually more behind the image.
$endgroup$
– EKons
Mar 26 at 15:54












$begingroup$
@EKons After the black pawn captures en passant, it is on c3 for the white king to capture.
$endgroup$
– noedne
Mar 26 at 15:56




$begingroup$
@EKons After the black pawn captures en passant, it is on c3 for the white king to capture.
$endgroup$
– noedne
Mar 26 at 15:56


















draft saved

draft discarded




















































Thanks for contributing an answer to Puzzling Stack Exchange!


  • Please be sure to answer the question. Provide details and share your research!

But avoid



  • Asking for help, clarification, or responding to other answers.

  • Making statements based on opinion; back them up with references or personal experience.


Use MathJax to format equations. MathJax reference.


To learn more, see our tips on writing great answers.




draft saved


draft discarded














StackExchange.ready(
function () {
StackExchange.openid.initPostLogin('.new-post-login', 'https%3a%2f%2fpuzzling.stackexchange.com%2fquestions%2f81057%2fthere-is-only-s%25cc%25b6i%25cc%25b6x%25cc%25b6t%25cc%25b6y-one-place-he-can-be%23new-answer', 'question_page');
}
);

Post as a guest















Required, but never shown





















































Required, but never shown














Required, but never shown












Required, but never shown







Required, but never shown

































Required, but never shown














Required, but never shown












Required, but never shown







Required, but never shown







Popular posts from this blog

Plaza Victoria

In PowerPoint, is there a keyboard shortcut for bulleted / numbered list?

How to put 3 figures in Latex with 2 figures side by side and 1 below these side by side images but in...